Tag Archives: Dutuyen

Đề thi và đáp án chọn đội dự tuyển PTNK năm 2022

Thời gian làm bài 120 phút.

Bài 1. Cho $a, b, c \geq 0$ thỏa $a^2+b^2+c^2=1$. Tìm giá trị lớn nhất và giả trị nhỏ nhất của biểu thức $P=a b+b c+c a-2(a+b+c)$.

Bài 2. Cho $k, n \in Z^{+}$, có bao nhiêu đơn ánh từ $\{1, 2, \cdots, 2k+1\} \to \{1, 2, \cdots, 2n\}$ thỏa $f(1) < f(2) < \ldots < f(k) < f(k+1) > f(k+2)>\ldots> f(2 k)>f(2 k+1)$ và $f(k+1) \neq 2 n-2$.

Bài 3. Cho $n$ là số nguyên dương, kí hiệu $a(n)=1+2+\ldots+n$ và $b(n)=1^2+2^2+\ldots+n^2$. Hỏi có tồn tại số $n$ sao cho $2(n+1) a(n)+3 b(n)-1$ là số chính phương?

Bài 4. Cho tam giác $A B C$ có $2 A=5 B=10 C$. Phân giác trong $B D$ cẳt trung tuyển $C M$ tại I. Một đường thẳng $d$ đi qua $D$ vuông góc với $A C$ cắt $B C$ và $A I$ lần lượt tại $E$ và $K . A E$ cắt $C K$ tại $F$. Chứng minh: $M F$ song song $B K$.

Lời giải tham khảo

Bài 1. Đặt $t=a+b+c$ ta có $a(1-a) \geq 0, b(1-b) \geq 0, c(1-c) \geq$, suy ra $a+b+c \geq$ $a^2+b^2+c^2=1$, và $(a+b+c)^2 \leq 3\left(a^2+b^2+c^2\right)=3$, suy ra $a+b+c \leq \sqrt{3}$ Ta có $1=(a+b+c)^2-2(a b+b c+a c) \Rightarrow a b+b c+c a=\frac{t^2-1}{2}$.
Do đó $P=\frac{t^2-1}{2}-2 t=\frac{1}{2} t^2-2 t-\frac{1}{2}$ với $1 \leq t \leq \sqrt{3}$.
Khảo sát hàm bậc hai trong đoạn ta có $\max P=-2$ khi $t=1$ và $\min P=1-2 \sqrt{3}$.
Vậy $\max P=-2$ khi $a=1, b=c=0$ và min $P=1-2 \sqrt{3}$ khi $a=b=c=\frac{1}{\sqrt{3}}$.

Bài 2. Do đó $f$ là đơn ánh, $\operatorname{Im} f$ là một tập con có $2 k+1$ phần tử của $A$, mặt khác $f(k+1)$ là giá trị lớn nhất nên $\operatorname{Im} f$ có giá trị lớn nhất khác $2 n-2$.
Ta đếm số tập con có $2 k+1$ phần tử của $A$ mà phần tử lớn nhất khác $2 n-2$. Số tập con có $2 k+1$ của $A$ là $C_{2 n}^{2 k+1}$, số tập con có $2 k+1$ mà có phần tử lớn nhất $2 n-2$ là bằng với số tập con có $2 k$ phần tử của ${1,2, \cdots 2 k-3}$, là $C_{2 n-3}^{2 k}$.
Do đó theo nguyên lí bù trừ số tập con có $2 k+1$ của tập $A$ mà phần tử lớn nhất khác $2 n-2$ là $\left(C_{2 n}^{2 k+1}-C_{2 n-3}^{2 k}\right)$.
Tiếp theo ta đếm số đơn ánh từ ${1,2, \cdots, 2 k+1}$ tới $A^{\prime}=\left\{a_1, a_2, \cdots, a_{2 k+1}\right\}$ thỏa đề bài, ta có $f(k+1)=a_{2 k+1}$, nên số đơn ánh bằng số cách chọn $k$ phần tử từ $A^{\prime}$ nên bằng $C_{2 k}^k$.
Vậy số đơn ánh thỏa đề bài $C_{2 k}^k\left(C_{2 n}^{2 k+1}-C_{2 n-3}^{2 k}\right)$

Bài 3. Ta có $a(n)=\frac{n(n+1)}{2}, b(n)=\frac{n(n+1)(2 n+1)}{6}$
Khi đó $P(n)=2(n+1) a(n)+3 b(n)-1=\frac{n(n+1)(4 n+3)}{2}-1$.
Giả sử $P(n)$ là số chính phương ta có $n(n+1)(4 n+3)=2\left(x^2+1\right)$, ta có $n(n+1)(4 n+3)$ luôn có ước nguyên tố dạng $p=4 k+3$, suy ra $p \mid 2\left(x^2+1\right)$ suy ra $p|x, p| 1$, vô lí! Vậy không tồn tại $n$ để $P(n)$ là số chính phương.

Bài 4.

Ta tính được $\angle A=\frac{5 \pi}{8}, \angle B=\frac{\pi}{4}, \angle C=\frac{\pi}{8}$. Vẽ đường cao $A N, N$ thuộc $B C$.
Ta có $\frac{B N}{N C}=\frac{A N}{N C}=\frac{\sin C}{\cos C}$ và $\frac{A D}{C D}=\frac{A B}{B C}=\frac{\sin C}{\sin 5 C}, \sin 5 C=\cos C$, suy ra $\frac{B N}{N C}=\frac{A D}{C D}$, do đó $A N, B D, C M$ đồng quy tại $I$ và $D N | A B$.
Ta có $\angle B A N=\angle A N D=\angle A C K=2 \angle A C K$, suy ra $A C K$ cân và $N$ là trung điểm $A K$, từ đó tam giác $A B K$ vuông cân.
Khi đó $\angle F N K=\angle A C K=45^{\circ}=\angle A K B$ và $\angle A N M=45^{\circ}$, do đó $M, N, F$ thẳng hàng và $M F | B K$.

Đề thi và đáp án chọn đội dự tuyển 10 trường PTNK năm 2023

Bài 1. Cho ba số thực $a, b, c>0$ thỏa mãn $a+b+c=3$.
(a) Chứng minh rằng $(a b+b c+c a)(a b c+1) \geq 6 a b c$.
(b) Tìm số nguyên dương k lớn nhất sao cho $a b c\left(a^k+b^k+c^k\right) \leq 3$.

Bài 2 .Với mỗi số thực $x,[x]$ gọi là phần nguyên của $x$ – là số nguyên lớn nhất không vượt quá $x$ và ${x}:=x-[x]$ gọi là phần lẻ của $x$.
Cho $p$ là số nguyên tố lẻ, chứng minh rằng với mọi số nguyên dương $k$ nhỏ hơn $p$ thì tổng $$S=\left\{\frac{k}{p}\right\}+\left\{\frac{2 k}{p}\right\}+\left\{\frac{3 k}{p}\right\}+\ldots+\left\{\frac{(p-1) k}{p}\right\}$$ không đổi. Tính S.

Bài 3. Cho tam giác $A B C$ nôi tiếp đường tròn $(\omega)$, tiếp tuyến của $(\omega)$ tai $\mathrm{B}$ là $d_1$, tai $\mathrm{C}$ là $d_2$. I là điểm thuôc trung trự $\mathrm{BC}$, đường tròn tâm $\mathrm{I}$ bán kính $\mathrm{IB}$ cắt các canh $\mathrm{AB}, \mathrm{AC}$ tại $\mathrm{D}, \mathrm{E}$. $\mathrm{CD}$ cắt $d_1$ tai $\mathrm{F}, \mathrm{BE}$ cắt $d_2$ tai $\mathrm{G}$ sao cho $\mathrm{F}, \mathrm{G}$ cùng phía $\mathrm{A}$ so với $\mathrm{BC}$. Đường tròn ngoai tiếp tam giác $\mathrm{BDF}$ cắt $\mathrm{BE}$ tại $\mathrm{K}$, đường tròn ngoại tiếp tam giác CEG cắt $\mathrm{CD}$ tại L.
(a) Khi $\mathrm{I}$ thuộc $\mathrm{BC}$, gọi $\mathrm{P}$ là giao điểm của $\mathrm{FK}$ và $\mathrm{GL}$. Chứng minh $\mathrm{AP}$ đi qua tâm của $(\omega)$.
(b) Khi I khác phía $\mathrm{A}$ đối với $\mathrm{BC}, \mathrm{DE}$ cắt $d_1$ tại $\mathrm{R}, d_2$ tại $\mathrm{S}$. Đường tròn ngoại tiếp tam giác ISR cắt $\mathrm{BC}$ tại $\mathrm{X}, \mathrm{Y}$. Chứng minh $B X=C Y$.

Bài 4 Tìm số nguyên dương $s$ lớn nhất thỏa mãn tính chất sau: Với mọi bộ số nguyên dương nhỏ hơn hay bằng 10 (không nhất thiết phân biệt) có tồng bằng $s$ ta luôn có thể chia thành hai nhóm mà tổng các số thuộc mỗi nhóm nhỏ hơn hay bằng 70 .

Lời giải

Bài 1.

(a) Đặt $a=\min {a, b, c}$, suy ra $a \leq 1$.
Khi đó $(a-1)^3 \leq 1 \Rightarrow a^3-3 a^2+3 a-1 \leq 0 \Rightarrow \frac{1}{a}+a(3-a) \geq 3$, suy ra $\frac{1}{a}+a b+a c \geq 3$, hơn nữa
$$
\frac{1}{b}+\frac{1}{c}+b c \geq 3 \sqrt[3]{\frac{1}{b} \frac{1}{c} b c}=3
$$
Từ đó $\frac{1}{a}+\frac{1}{b}+\frac{1}{c}+a b+b c+a c \geq 6$. hay $(a b+b c+c a)(a b c+1) \geq 6 a b c$.
(b) Cho $a=2, b=c=\frac{1}{2}$, suy ra $k<3$, ta chứng minh $k=2$ thì bất đẳng thức thỏa với mọi $a, b, c$ thỏa điều kiện, thật vậy

$a b c\left(a^2+b^2+c^2\right) =\frac{1}{3} \cdot a b c(a+b+c)\left(a^2+b^2+c^2\right) $
$\leq \frac{1}{9} \cdot(a b+b c+c a)^2 \cdot\left(a^2+b^2+c^2\right) $
$=\frac{1}{9} \cdot(a b+b c+c a)(a b+b c+c a)\left(a^2+b^2+c^2\right) $
$ \leq \frac{1}{9} \cdot \frac{1}{27} \cdot\left(a^2+b^2+c^2+2(a b+b c+c a)\right)^3 $
$ =\frac{1}{9} \cdot \frac{1}{27} \cdot 3^6=3$

Bài 2.

Với $p$ nguyên tố lẻ thì $(k, p)=1$ với mọi $0<k<p$. Ta chứng minh $p-1$ số $k, 2 k, \cdots,(p-1) k$ là hệ thặng dư thu gọn của $p$, thật vậy, giả sử $i k \equiv j k($ $\bmod p)$ với $i, j<p$ thì $k(i-j) \equiv 0(\bmod p)$, suy ra $i=j$.
Khi đó $S=\left\{\frac{k}{p}\right\}+\left\{\frac{2 k}{p}\right\}+\left\{\frac{3 k}{p}\right\}+\ldots+\left\{\frac{(p-1) k}{p}\right\}=\frac{1}{p}+\frac{2}{p}+\cdots \frac{p-1}{p}=$ $\frac{p-1}{2}$ không đổi.

Bài 3.

(a) Gọi $O$ là tâm của $\omega$. Ta có $\angle S D B=\angle A D E=\angle A C B=\angle S B D$ nên $\triangle S B D$ cân tại $S$. Tương tự $\triangle R E C$ cân tại $R$. Biến đổi góc
$$
\angle K F L=\angle K F D=\angle K B D=\angle D C E=\angle E G L \angle K G L,
$$
suy ra $F, K, L, G$ đồng viên.
Do $I \in B C$ nên $\angle B D C=90^{\circ}$, mà $\triangle S B D$ cân tại $S$ nên $S$ là tâm đường tròn $(F D K)$. Tương tự, $R$ là tâm đường tròn $(G E L)$. Ta có
$$
A D \cdot A B=A E \cdot A C, \quad P K \cdot P F=P L \cdot P G,
$$
suy ra $A P$ là trục đẳng phương của hai đường tròn $(F D K)$ và $(G E L)$, do đó $A P \perp R S$.
Mà $A O \perp D E$ nên $A, O, P$ thằng hàng.

(b) Gọi $M, N$ lần lượt là giao điểm của $I S, I R$ với $B C . \triangle S B D$ cân tại $S$ nên suy ra $I S$ là đường trung trực của $B D$, tương tự $I R$ là đường trung
Tập san Toán học STAR EDUCATION
trực của $E C$. Biến đổi góc
$$
\begin{aligned}
& \angle M S D=90^{\circ}-\angle S D B=90^{\circ}-\angle A D E=90^{\circ}-\angle A C B=\angle C N G . \
\Rightarrow & \angle I S R=\angle Y N G \Rightarrow \angle I S Y+\angle Y S R=\angle M Y I+\angle Y I R \Rightarrow \angle I S Y= \
& \angle X Y I=\angle X S I .
\end{aligned}
$$
Vậy $S I$ là tia phân giác của $\angle X S Y$ nên $I$ nằm trên đường trung trực của $X Y$. Mà $I$ cũng nằm trên đường trung trực của $B C$ nên $B X=C Y$.

Bài 4.

Ta chứng minh rằng $s=133$ là số lớn nhất thoả mãn điều kiện bài toán. Trước hết, giả sử rằng $s$ là một số thoả mãn điều kiện đã cho.

Viết $s=9 k+r (k, r \in \mathbb{Z}{\geq 0}, 1 \leq r \leq 9 )$.

Nếu $s \geq 134$, xét một bộ số gồm $k$ số 9 và số còn lại bằng $s-9 k$. Trong bộ số này có không quá một số khác 9 nên khi chia chúng thành hai phần khác rỗng, phải có ít nhất một bộ chứa toàn số 9. Hơn nữa, $$ 9 \cdot 7=63<70<9 \cdot 8 $$ nên bộ số này có tổng tối đa là 63 . Nhưng khi đó tổng của các số còn lại, gọi là $T$, sẽ phải thoả mãn $$ T \geq 134-63=71>70 $$ vô lý do $T \leq 70$. Từ đó phải có $s \leq 133$. Bây giờ ta chứng minh rằng $s=133$ thoả mãn điều kiện bài toán. Trước hết, ta chứng minh rằng với mọi bộ số nguyên dương không vượt quá 10 có tổng bằng 133, khi chia thành hai phần khác rỗng là $X, Y$ khác rống (có thể có các phần tử trùng nhau), sao cho

$$ M=\sum{x \in X} x-\sum_{y \in Y} y \geq 0$$

và $M$ nhỏ nhất có thể, thì $M \leq 8$. Thật vậy, giả sử rằng $M \geq 9$ thì
$$
\sum_{x \in X} \geq \frac{1}{2}\left(\sum_{x \in X} x+\sum_{y \in Y} y+9\right) \geq \frac{133+9}{2}=71 .
$$
Vì mỗi phần tử của $X$ không vượt quá 10 nên $X$ có ít nhất 8 phần tử. Đặt $t=\min X$. Xét hai tập hợp
$$
\left\{\begin{array}{l}
X^{\prime}=X \cup{t} \
Y^{\prime}=Y \backslash{t}
\end{array}\right.
$$
thì $X^{\prime}, Y^{\prime} \neq \emptyset$, đều gồm các số nguyên dương không vượt quá 10 , và có tổng bằng 133. Vì tính nhỏ nhất của $M$ nên
$$
M \leq\left|\sum_{x \in X^{\prime}} x-\sum_{y \in Y^{\prime}} y\right|=\left|\sum_{x \in X} x-\sum_{y \in Y} y-2 t\right|=|M-2 t|
$$
Kết hợp với $M \geq 9$ và $1 \leq t \leq 10$ thì $9 \leq M \leq t \leq 10$. Có hai khả năng sau:

  • Nếu $M=10$ thì
    $$
    \sum_{x \in X} x=\frac{133+10}{2} \notin \mathbb{Z}
    $$
    là một điều vô lý.
  • Nếu $M=9$ thì
    $$
    \sum_{x \in X} x=\frac{133+9}{2}=71 .
    $$
    Nếu $t=9$ thì $X$ gồm toàn số 9 và số 10 , nên có thể viết được
    $$
    71=9 k+10 l\left(k, l \in \mathbb{Z}{\geq 0}\right) . $$ Do đó $9 k \equiv 1(\bmod 10)$, dẫn đến $k \equiv 9(\bmod 10)$ và $k \geq 9$. Hệ quả là $$ 9 k+10 l \geq 9 k \geq 81>71 $$ cũng là điều vô lý. Từ đó điều giả sử là sai hay phải có $M \leq 8$, dẫn đến $$ \sum{y \in Y} y \leq \sum_{x \in X} x \leq \frac{1}{2}\left(\sum_{x \in X}+\sum_{y \in Y} y+8\right)=\frac{133+8}{2} .
    $$
    Nhưng các tổng là số nguyên nên
    $$
    \sum_{y \in Y} y \leq \sum_{x \in X} x \leq 70,
    $$
    nghĩa là cách chia $(X, Y)$ thoả mãn điều kiện bài toán. Tóm lại, $s=133$ là số lớn nhất thoả mãn yêu cầu đề bài. Bài toán kết thúc.

Đáp án đề thi chọn đội dự tuyển trường PTNK năm 2020

Thời gian làm bài 120 phút

Đề bài.

Bài 1. Tìm giá trị nhỏ nhất của biểu thức $P=\frac{a^{4}+b^{4}+2}{\left(a^{2}-a+1\right)\left(b^{2}-b+1\right)}$, với $a, b \in \mathbb{R}$.
Bài 2. Tìm tất cả các hàm $f: \mathbb{Q}^{+} \rightarrow \mathbb{Q}^{+}$thỏa mãn
$$
f\left(x^{2} f(y)^{2}\right)=f(x)^{2} f(y), \text { với mọi } x, y \in \mathbb{Q}^{+} .
$$
Bài 3. Cho $x_{1}, x_{2}, x_{3}, \ldots$ là dãy số nguyên thỏa mãn đồng thời hai điều kiện $1=$ $x_{1}<x_{2}<x_{3} \ldots$ và $x_{n+1} \leq 2 n$ với $n=1,2,3 \ldots$ Chứng minh rằng với mọi số nguyên dương $k$, tồn tại các số nguyên $i>j$ sao cho $x_{i}-x_{j}=k$.

Bài 4. Cho tam giác $A B C$ cân tại $A$, nội tiếp đường tròn tâm $O$ bán kính $R$. Gọi $M$ là điểm trên cạnh $A B$ sao cho $\overrightarrow{A M}=\frac{1}{3} \overrightarrow{A B}$. Đường tròn tâm $M$ bán kính $M B$ cắt đường tròn tâm $O$ tại điểm thứ hai là $D$. Một đường thẳng qua $M$ song song với $A D$ cắt $A C$ tại $N$. Chứng minh rằng $\overrightarrow{A N}=\frac{2}{3} \overrightarrow{A C}$.

Đáp án

Bài 1. Với mọi $x \in \mathbb{R}$, ta có
$$
x^{4}+1-\frac{2}{9}\left(x^{2}-x+1\right)^{2}=\frac{1}{9}(x+1)^{2}\left(7 x^{2}-10 x+7\right) \geq 0 .
$$
Vì thế nên ta có
$$
P \geq \frac{2}{9} \frac{\left(a^{2}-a+1\right)^{2}+\left(b^{2}-b+1\right)^{2}}{\left(a^{2}-a+1\right)\left(b^{2}-b+1\right)}=\frac{2}{9}\left(\frac{a^{2}-a+1}{b^{2}-b+1}+\frac{b^{2}-b+1}{a^{2}-a+1}\right) \geq \frac{4}{9} .
$$
Suy ra giá trị nhỏ nhất của $P$ là $\frac{4}{9}$, đạt được khi $a=b=-1$.

Bài 2. Giả sử $f$ là một hàm thỏa mãn các yêu cầu của bài toán. Đặt $f(1)=a>$ 0 , trong phương trình đề cho, thay $x=y=1$ ta có $f\left(a^{2}\right)=a^{3}$.
Từ đó, tiếp tục lần lượt thay $x$ bởi $a^{2}, y$ bởi 1 và $x$ bởi $1, y$ bởi $c^{2}$ vào phương trình ấy, ta thu được
$$
a^{7}=f\left(a^{6}\right)=a^{5} .
$$
Chú $\hat{y} a>0$ nên ta có $a=1$, tức $f(1)=1$. Thay $x$ bởi 1 vào phương trình đề cho, ta có
$$
f\left(f(y)^{2}\right)=f(y) \text {, với mọi } y \in \mathbb{Q}^{+} \text {. }
$$
Lại thay $y$ bởi 1 vào phương trình đề cho, ta có
$$
f(x)^{2}=f\left(x^{2}\right), \text { với mọi } x \in \mathbb{Q}^{+} .
$$
Suy ra
$$
f(x)=f\left(f(x)^{2}\right)=f(f(x))^{2}=\ldots=f^{n+1}(x)^{2^{n}}, \text { với mọi } x \in \mathbb{Q}^{+},
$$
trong đó $f^{n+1}(x)$ là $n+1$ lần tác động $f$ vào $x$. Từ đó, nếu tồn tại $q \in \mathbb{Q}^{+}$sao cho tồn tại $p \in \mathbb{P}$ thỏa mãn $v_{p}(f(q)) \neq 0$ thì ta có
$$
v_{p}(f(q))=v_{p}\left(f^{n+1}(q)^{2^{n}}\right)=2^{n} v_{p}\left(f^{n+1}(q)\right) \neq 0 .
$$
Trong đẳng thức trên, cho $n \rightarrow+\infty$ ta thấy điều vô lý. Suy ra $v_{p}(f(q))=0$ với mọi $q \in \mathbb{Q}^{+}, p \in \mathbb{P}$, hay $f(x) \equiv 1$.
Thử lại, ta kết luận $f(x) \equiv 1$ là hàm duy nhất thỏa mãn yêu cầu bài toán.

Bài 3. Với $k$ nguyên dương, ta xét $k+1$ số hạng của dãy là $x_{1}, x_{2}, \ldots, x_{k+1}$. Ta có $x_{1}=1 \leq k$, gọi $q$ là số lớn nhất thỏa mãn $x_{q} \leq k$ thì ta có $q<k+1$ và
$$
1 \leq x_{1}<x_{1}<\cdots<x_{q} \leq k<x_{q+1}<\cdots<x_{k+1}<2 k \text {. }
$$
Nếu tồn tại $1 \leq j<i \leq k+1$ sao cho $x_{i}-x_{j}=k$ thì ta có ngay điều cần chứng minh. Ngược lại, ta có các số
$$
x_{1}+k, x_{2}+k, \ldots, x_{q}+k, x_{q+1}, \ldots, x_{k+1}
$$
là $k+1$ số nguyên đôi một phân biệt, tất cả đều lớn hơn $k$ nhưng lại không vượt quá $2 k$, vô lí!

Từ đó suy ra với mọi $k$ nguyên dương, luôn tồn tại các số nguyên $i>j$ sao cho $x_{i}-x_{j}=k$.

Bài 4. Ta có $O B=O D, M B=M D$ nên dễ thấy $O M$ là phân giác ngoài của góc $A M D$, mà $O A=O D$ nên suy ra $O \in(A M D)$.

Gọi $N^{\prime}$ là giao điểm khác $A$ của $(A M D)$ và $A C$. Ta chứng minh $N$ trùng $N^{\prime}$. Thật vậy, ta có $\overrightarrow{A M}=\frac{1}{3} \overrightarrow{A B}$ nên $\angle A M O$ tù, do đó nếu $N^{\prime}$ nằm ngoài tia $A C$ thì $N^{\prime}$ nằm khác phía $O$ so với $A M$ nên
$$
\angle A M O=\angle A N^{\prime} O=\angle C A O-\angle A O N^{\prime}<\angle C A O<90^{\circ},
$$
vô lý. Suy ra $N^{\prime}$ nằm trên tia $A C$, kéo theo $A O$ là phân giác trong góc $M A N^{\prime}$ nên $O M=O N^{\prime}$, mà $O A=O D$ nên $M N^{\prime}$ song song $A D$, suy ra $N$ trùng $N^{\prime}$.

Từ đó, dễ thấy $A M N D$ là hình thang cân nên $A N=M D=M B$, hơn nữa $N$ nằm trên tia $A C$ nên ta thu được
$$
\overrightarrow{A N}=\frac{2}{3} \overrightarrow{A C}
$$
Ta có điều cần chứng minh.

Tài liệu tham khảo

[1] Nguyễn Tăng Vũ, Lê Phúc Lữ, Nguyễn Tiến Hoàng, Đề thi và đáp án kì thi dự tuyển và đội tuyển PTNK 2008-2021

Đề và đáp án thi chọn đội dự tuyển lớp 10 năm 2012 – 2013

Bài 1. Cho $a,b$ là hai số thực thoả mãn $a+b\ge 0$. Chứng minh rằng:

$$\left(\dfrac{a^2+b^2}{2}\right)^3\ge 4(a^3+b^3)(ab-a-b).$$

Bài 2. Tìm tất cả các số nguyên dương $m,n$ để $\dfrac{5mn+5m}{3m^2+2n^2}$ là số nguyên.

Bài 3.  Cho tập hợp $X={1,2,\ldots,2n-1}$ gồm $2n-1$ số tự nhiên $(n\ge 2)$. Tô màu ít nhất $n-1$ phần tử của $X$ với điều kiện sau: nếu $a,b\in X$ (không nhất thiết phân biệt) được tô màu thì $a+b$ cũng được tô màu, miễn là $a+b\in X$. Gọi $S$ là tổng tất cả các số không được tô màu của $X$.

a/Chứng minh rằng $S\le n^2$.

b/Chỉ ra tất cả các phép tô màu sao cho $S=n^2$.

Bài 4. Cho đường tròn $(O)$ và dây cung $AB$ cố định khác đường kính. Gọi $C$ là điểm chính giữa cung lớn $AB$. Đường thẳng $d$ thay đổi qua $C$ cắt tiếp tuyến tại $A$ và tiếp tuyến tại $B$ của $(O)$ lần lượt tại $D,E$. Gọi $Q$ là giao điểm của $AE$ và $BD$. Chứng minh rằng đường thẳng $PQ$ luôn đi qua một điểm cố định khi $d$ thay đổi.

Giải

Bài 1. Ta xét các trường hợp sau:

  •  Nếu $ab<0$, ta có vế trái dương và vế phải âm nên bất đẳng thức đúng.
  •  Nếu $ab \ge 0$, kết hợp với $a+b \ge 0$, ta suy ra $a,b \ge 0.$

Áp dụng lần lượt các đánh giá $4xy \le (x+y)^2$ và $2xy \le x^2 + y^2$ thì:

$$\begin{align*} 4(a^3+b^3)(ab-a-b) & = 4(a+b)(ab-a-b)(a^2-ab+b^2) \\ & \leq a^2b^2(a^2-ab+b^2) \\ & \leq \dfrac{ab(a^2+b^2)^2}{4} \end{align*}$$

Mà ta có:

$$\dfrac{(a^2+b^2)^3}{8}=\dfrac{(a^2+b^2)^2}{4}.\dfrac{a^2+b^2}{2}\geq \dfrac{ab(a^2+b^2)^2}{4}.$$

Từ hai đánh giá trên, ta có đpcm.

Bài 2.

Đặt $k=\dfrac{5mn+5m}{3m^2 + 2n^2} \in \mathbb{N}^*$. Suy ra

$$3km^2 – 5(n+1)m + 2kn^2 = 0$$ là một phương trình theo ẩn $m$ với

$$\Delta = 25(n+1)^2 – 24k^2n^2 = (25-24k^2)n^2 + 50n + 25 \ge 0. (*)$$

Xét các trường hợp sau:

  • Nếu $k>1$, ta có:

$\Delta _1′ = 625 – 25\left( {25 – 24{k^2}} \right) = 600{k^2} > 0$, mà $25 – 24k^2 < 0$.

Suy ra bất phương trình $(*)$ có nghiệm khi $n \le \dfrac{25+10k\sqrt{6}}{24k^2-25}< 2$ (dễ dàng chứng minh).

Vì thế nên $n=1$ (do $n \in \mathbb{N^{*}}$). Ta có:

$$ \begin{aligned} k= \dfrac{10m}{3m^2 + 2} \in \mathbb{N^{*}} & \Rightarrow \dfrac{30m^2}{3m^2 + 2} \in \mathbb{N^{*}} \Rightarrow \dfrac{-20}{3m^2 + 2} \in \mathbb{N^{*}} \\ & \Rightarrow 3m^2 +2 \in \left\{ {2;5;10;20} \right\} \text{ vì } 3m^2+2 \ge 2, \forall m \\ & \Rightarrow m=1 \text{ do } m \in \mathbb{N^{*}}. \end{aligned} $$

Thử lại ta nhận $(m;n)=(1;1)$

  •  Nếu $k=1$ thì $\Delta = n^2 + 50n +25 = x^2$ ($x \in \mathbb{N}$) nên suy ra $$(n+x+25)(n-x+25) = 600.$$

Từ đây với lưu ý $n+x+25 > n-x+25 > 0, n+x+25 > 25$ ta có $$n \in \left\{ {126;52;28;10;6} \right\}.$$ Thay vào phương trình đầu, ta tìm được  $$(m;n)=(9;6),(5;10),(32;28),(32;52),(81;126).$$

Bài 3.

(a) Rõ ràng nếu $1$ được tô thì tất cả các số cũng sẽ được tô, kéo theo $S=0 \le n^2$, thỏa mãn. Do đó, ta chỉ cần xét $1$ không được tô. Gọi các số được tô là $$1 < a_1 < a_2 < \ldots < a_m \le 2n-1,$$

trong đó $m \ge n-1$. Ta sẽ chứng minh rằng với mọi $k$ mà $1 \le k \le m/2$ thì

$$a_k + a_{m-k+1} \ge 2n.$$

Giả sử ngược lại rằng $a_k+a_{m-k+1} <2n$ thì tổng hai số trên phải là số được tô màu. Do đó, nó phải thuộc tập hợp

$$Q = \left\{ {{a_{m – k + 2}};{a_{m – k + 3}};\ldots;{a_m}} \right\}.$$

Mặt khác lại xét chỉ số $i < k$ thì rõ ràng do dãy đang xét là tăng nên ta cũng có tổng ${a_i} + {a_{m – k + 1}}$ thuộc tập hợp $Q$ ở trên. Suy ra $|Q| \ge k,$ mâu thuẫn vì rõ ràng $Q$ chỉ có $k-1$ phần tử. Vì thế nên ta phải có $a_k + a_{m-k+1} \ge 2n.$

Đến đây, ta có ${a_k} + {a_{m – k + 1}} \ge 2n$ với mọi $k \in \left\{ {1;2;3;\ldots;m} \right\}$ nên

$$\sum\limits_{i = 1}^m {{a_i} = \frac{1}{2}} \sum\limits_{i = 1}^m {({a_i} + {a_{m – i + 1}}) \geqslant n(n – 1)}, \text{ suy ra }$$

$$S = \sum\limits_{i = 1}^{2n – 1} i – \sum\limits_{i = 1}^m {{a_i} \leqslant n(2n – 1) – n(n – 1) = {n^2}}.$$

(b) Để có $S=n^2$ thì dấu bằng xảy ra ở tất cả các đánh giá trên, tức là ta tô được đúng $m=n-1$ số và $a_k+a_{n-k}=2n$ với mọi $1 \le k \le n-1.$

Ta có $(2{{a}_{1}},{{a}_{1}}+{{a}_{2}},{{a}_{1}}+{{a}_{3}},\ldots ,{{a}_{1}}+{{a}_{n-2}})$ là một hoán vị của các số $({{a}_{2}},{{a}_{3}},\ldots ,{{a}_{n-1}}).$

Do tính tăng của hai dãy này nên ta có $$2{{a}_{1}}={{a}_{2}},{{a}_{1}}+{{a}_{2}}={{a}_{3}},\ldots ,{{a}_{1}}+{{a}_{n-2}}={{a}_{n-1}}.$$ Vì thế nên ${{a}_{k}}=k{{a}_{1}}$ với mọi $1\le k\le n-1.$ Mà $2n={{a}_{1}}+{{a}_{n-1}}=n{{a}_{1}}$ nên ta có ${{a}_{1}}=2,$ từ đây tìm được các tô duy nhất là $(2,4,6,\ldots ,2n-2)$ thỏa mãn đề bài.

Bài 4.

Giả sử $AD\cap BE=T,AB\cap DE=I$ và $TQ$ cắt $DE,AB$ lần lượt ở $X,S.$ Khi đó dễ thấy rằng

$(IX,DE)=(IS,AB)=-1.$

Mà $PI$ đi qua trung điểm cung lớn $AB$ của $(O)$ nên $PI$ là phân giác ngoài, kéo theo $PS$ là phân giác trong nên nó đi qua $N$ là trung điểm cung nhỏ $AB$ của $(O)$.

Gọi $M$ là trung điểm $AB.$ Theo tính chất phương tích thì $TN\cdot TC=T{{A}^{2}}=T{{B}^{2}}=TM\cdot TO$, mà $O$ là trung điểm $CN$ nên theo hệ thức Maclaurin thì $(TM,NC)=-1.$

Không có mô tả.

 

Lại có $(TQ,XS)=-1$ nên chùm $P(XS,QT)=-1$, mà $PX$ đi qua $C,$ $PS$ đi qua $N$ nên ta phải có $PQ$ đi qua $M$ là điểm cố định.

Nhận xét: Bài toán có thể xử lý theo hướng tự nhiên hơn bằng cách dùng định lý Ceva sin. Từ kết quả trên, ta còn thấy được rằng nếu lấy $CQ$ cắt $AB$ ở $K$ thì $PK$ là đối trung của tam giác $PAB,$ kéo theo $P,K,T$ thẳng hàng.

Đáp án đề thi chọn đội dự tuyển lớp 10 năm 2016 – 2017

Bài 1: Cho $x,y,z$ là các số thực dương thoả mãn $x+y+z=1$. Chứng minh rằng:

$$\dfrac{x^4}{x^3+y^2+z^2}+\dfrac{y^4}{y^3+z^2+x^2}+\dfrac{z^4}{z^3+x^2+y^2}\ge \dfrac{1}{7}.$$

Bài 2: Tìm tất cả các hàm số $f:\mathbb N^* \rightarrow \mathbb N^*$ thoả mãn đồng thời các điều kiện:

i/ $f(mn)=f(m)f(n)\ \forall m,n \in \mathbb N^*$.

ii/ $f(m)+f(n)$ chia hết cho $m+n$ $\forall m,n \in \mathbb N^*$.

iii/ $f(2017)=2017^3$.

Bài 3. Cho đường tròn $(O)$ và dây cung $AB$ cố định. $C$ là một điểm thay đổi trên cung lớn $AB$ sao cho tam giác $ABC$ nhọn. Gọi $I,I_a,I_b$ lần lượt là tâm đường tròn nội tiếp, tâm đường tròn bàng tiếp $\angle BAC$ và $\angle ABC$ của tam giác $ABC$.

a/ Gọi $M$ đối xứng với $I$ qua $O$. Chứng minh rằng tam giác $MI_{a}I_{b}$ cân.

b/ Gọi $H,K$ lần lượt là hình chiếu của $I_a,I_b$ trên $OI$. Đường thẳng qua $H$ vuông góc với $BI_a$ và đường thẳng qua $K$ vuông góc với $AI_b$ cắt nhau tại $P$. Chứng minh rằng $P$ thuộc một đường cố định khi $C$ thay đổi.

Bài 4. Cho $S$ là tập hợp khác rỗng và $A_1,A_2,\ldots,A_m\ (m\ge 2)$ là $m$ tập con của $S$. Gọi $\mathcal T$ là tập hợp gồm tất cả các tập hợp $A_i\Delta A_j\ (1\le i,j \le m$). Chứng minh rằng $|\mathcal T| \ge m$.

(Ký hiệu $A\Delta B=(A\backslash B)\cup (B\backslash A)$ là hiệu đối xứng của hai tập hợp $A,B$).

Giải

Bài 1.

Theo bất đẳng thức Cauchy-Schwarz, ta có

$$ \sum \dfrac{x^4}{x^3+y^2+z^2} \ge \dfrac{ \left( x^2+y^2+z^2 \right)^2}{ x^3+y^3+z^3+2 \left( x^2+y^2+z^2 \right)} $$

Cần chứng minh $\dfrac{ \left( x^2+y^2+z^2 \right)^2}{ x^3+y^3+z^3+2 \left( x^2+y^2+z^2 \right)} \ge \dfrac{1}{7} $ hay

$$7 \left( x^2+y^2+z^2 \right)^2 \ge x^3+y^3+z^3+2 \left( x^2+y^2+z^2 \right).$$ Ta có ${{(xy+yz+zx)}^{2}}\ge 3xyz(x+y+z)=3xyz$ và

$${{x}^{3}}+{{y}^{3}}+{{z}^{2}}-3xyz=(x+y+z)({{x}^{2}}+{{y}^{2}}+{{z}^{2}}-xy-yz-zx)$$ nên ${{x}^{3}}+{{y}^{3}}+{{z}^{3}}=3xyz+1-3(xy+yz+zx)\le {{(xy+yz+zx)}^{2}}+1-3(xy+yz+zx).$

Đặt $q=xy+yz+zx$ thì vì ${{(x+y+z)}^{2}}\ge 3(xy+yz+zx)$ nên $q\le \frac{1}{3}.$ Ta đưa về

$$7{{(1-2q)}^{2}}\ge {{q}^{2}}+1-3q+2(1-2q)$$ hay

$$(1-3q)(4-9q)\ge 0.$$

Do $q\le \frac{1}{3}$ nên $q\le \frac{4}{9}$ và bất đẳng thức trên là đúng. Vậy ta có đpcm.

Bài 2.

Nhận xét rằng vai trò của số $2017$ trong bài toán là không cần thiết cho nên ta sẽ giải bài toán khi thay $2017$ bởi số nguyên dương $p$ bất kỳ. Từ điều kiện đầu tiên, ta có được $f(p^k)=p^{3k}$ với $k$ là số nguyên dương bất kỳ.

Trong điều kiện thứ hai, thay $n$ bởi $m$, ta có $f(m)$ là bội của $m$ với mỗi $m$ nguyên dương nên ta đặt $f(m)=m.g(m)$ ($g:\mathbb{N^{*}}\rightarrow \mathbb{N^{*}}$). Khi đó ta có các điều kiện sau:

i/ $g(mn)=g(m).g(n) \forall m,n \in\mathbb{N^{*}}$

ii/ $mg(m)+ng(n)$ là bội của $m+n$.

iii/ $g(p^{n})=p^{2n} \forall n\in \mathbb{N^{*}}$.

Đặt $h(m)=g(m)-m^2$ ($h:\mathbb{N^{*}}\rightarrow \mathbb{Z}$) và thay $n$ bởi $p^n$ tại ii), ta có $m.h(m)$ là bội của $m+p^n$. Chọn $n$ đủ lớn thì $h(m)=0$ với mỗi $m$ hay $f(m)=m^3$ với mỗi $m$ nguyên dương. Thử lại thoả mãn.

Vậy $f(m)=m^3$ là nghiệm hàm duy nhất.

Bài 3.

(a) Trước hết, ta có một kết quả quen thuộc sau.

Bổ đề: Gọi $A_1$, $B_1$ lần lượt là điểm chính giữa các cung $BC$, $AC$ không chứa $A$, $B$ của $(O)$. Khi đó $A$, $I$, $A_1$, $I_a$ thẳng hàng và $A_1$ là trung điểm của $II_a$. Tương tự đối với $B$, $I$, $B_1$, $I_b$.

Trở lại bài toán, theo bổ đề, phép vị tự tâm $I$, tỉ số $2$ biến $\Delta OA_1B_1$ thành $\Delta MI_aI_b$, do đó tam giác này cân tại $M$.

Mở ảnh

(b) Ta thực hiện chuyển đổi mô hình. Gọi $I_a$ là tâm bàng tiếp góc $A$ của tam giác $ABC$ thì $(O)$ chính là đường tròn Euler của tam giác $I_aI_bI_c$. Xét bổ đề sau:

Bổ đề: Cho tam giác $ABC$ có đường thẳng $d$ đi qua tâm ngoại tiếp $O$. Gọi $D,E,F$ lần lượt là hình chiếu của $A,B,C$ lên $d$. Chứng minh rằng đường thẳng qua $D,E,F$ vuông góc với $BC,CA,AB$ đồng quy trên đường tròn $Euler$ của tam giác $ABC$.

Gọi $l$ là đường thẳng đi qua trực tâm $H$ của tam giác $ABC$ và vuông góc với $d$. Gọi $S$ là điểm anti-Steiner của $l$. $J$ là điểm đối xứng của $S$ qua $BC$ và $X$ là giao điểm của $SJ$ và $(O)$. $K$ là điểm đối xứng với $H$ qua $BC$. \medskip

Ta có: $$\angle AXS =\angle AKS=\angle KHJ$$ suy ra $HJ \parallel AX$. Do đó, $D$ nằm trên $AX$ hay $D$ là trung điểm $AX$. Suy ra đường thẳng qua $D$ vuông góc với $BC$ đi qua trung điểm $I$ của $SH$ và nằm trên đường tròn $Euler$ của tam giác $ABC$. \medskip

Trở lại bài toán, ta áp dụng bổ đề trên cho đường thẳng $OI$ đi qua tâm đường tròn $(I_aI_bI_c)$ thì dễ dàng có $P \in (O).$

Bài 4.

Ta sẽ chứng minh quy nạp theo $m$ cho điều này. Trước hết ta phát biểu bổ đề: $A\Delta B=A\Delta C$ thì $B=C$.

Giả sử $B\neq C$, khi đó không giảm tổng quát giả sử giả sử $a$ là phần tử thỏa $a\in B$ và $a\notin C$. Ta có hai trường hợp như sau:

  •  Nếu $a\in A$ khi đó $a\notin (A\setminus B),a\notin (B\setminus A)\Rightarrow a\notin A\Delta B$. Nhưng lại có $a\in (A\setminus C)$ nên suy ra $a\in A\Delta C$ nên $A\Delta B\neq A\Delta C$, vô lý.
  •  Nếu $a\notin A$ thì chứng minh tương tự suy ra $a\notin A\Delta C$ và $a\in A\Delta B$ nên suy ra $A\Delta B\neq A\Delta C$. Như vậy ta suy ra $B=C$.

Bây giờ ta sẽ quy nạp theo $m$. Với $m=1$ thì ta có một tập thuộc $T$ là tập rỗng. Với $m=2$ và hai tập $A,B$ thì ta có hai tập thuộc $T$ là tập rỗng và $A\Delta B$ thỏa. Như vậy giả thiết đúng với $m=1,2$.

Giả sử giả thiết đúng với $m=k$ thì ta chứng minh nó đúng với $m=k+1$. Xét $m+1$ tập $A_1,A_2,\ldots,A_{m+1}$. Nếu với $m$ tập $A_1,A_2,\ldots,A_m$ mà số lượng tập tạo thành không nhỏ hơn $m+1$ thì khi đó ta thêm vào một tập $A_{m+1}$ thì giả thiết vẫn đúng. Do đó ta chỉ xét cho trường hợp $|T|=m$.

Khi đó, nếu ta thêm vào một tập $A_{m+1}$ thì ta sẽ thêm vào tập $T$ các tập hợp $A_{m+1}\Delta A_1,\ldots,A_{m+1}\Delta A_{m+1}$. Nếu các tập này trùng với $m$ tập đã có trong $T$ thì do $|T|=m$ nên theo nguyên lý Dirichlet tồn tại $i,j,1\leq i<j\leq m+1$ để $A_{m+1}\Delta A_i=A_{m+1}\Delta A_j$ và theo bổ đề ta có $A_i=A_j$, vô lý. Vậy trong $m+1$ tập đó chắc chắn có một tập khác với các tập trong $T$ và số phần tử của $T$ tăng lên ít nhất một đơn vị, tức là $|T|\geq m+1$.

Vậy giả thiết quy nạp là đúng và ta có đpcm.

Đề thi chọn đội Dự Tuyển PTNK năm học 2020-2021

Kì thi chọn Dự tuyển trường Phổ thông Năng khiếu tham dự kì thi 30/04 được tổ chức vào tháng 01 năm 2021, đề gồm 4 bài, làm trong 120 phút.

Đề bài

Bài 1. Cho các số thực không âm $a, b, c$ thỏa mãn $a^{2}+b^{2}+c^{2}=1$. Tìm giá trị lớn nhất của biểu thức $$P=\frac{7}{2} a+(1-a)(\sqrt{a}+\sqrt{b}+\sqrt{c})+a^{2} b^{2} c^{2}$$

Bài 2. Tìm tất cả các hàm số $f: \mathbb{R} \rightarrow \mathbb{R}$ thỏa mãn $f(x-f(y))=4 f(x)+3 x+f(y)$ với mọi $x, y \in \mathbb{R}$.

Bài 3. Cho $n$ là số nguyên dương và $A=\left\{m \in \mathbb{N}^{*} \mid \operatorname{gcd}(m, 6)=1, m<30 n\right\}$ với $|A|=8 n+1$. Chứng minh rằng tồn tại 2 số phân biệt $a, b \in A$ sao cho $a \mid b$.
Bài 4. Cho điểm $M$ di động trên đường thẳng $d$ cố định và $O$ là điểm cố định nằm ngoài đường thẳng $d$. Gọi $A$ là hình chiếu của $O$ lên $d$, và $H$ là hình chiếu của $A$ trên $O M$. Gọi $D$ là trung điểm $H M$.
(a) Chứng minh rằng đường thẳng qua $H$, vuông góc với $A D$ luôn đi qua một điểm cố định. Gọi điểm đó là $N$.
(b) Chứng minh rằng tâm đường tròn $(H M N)$ luôn thuộc một đường thẳng cố định. Từ đó tính tỷ số $\frac{A M}{A O}$ để $(H M N)$ và $(O A H)$ tiếp xúc với nhau.

Đề thi và đáp án thi chọn đội dự tuyển trường PTNK năm học 2016-2017

ĐỀ BÀI

Bài 1. Cho $x,y,z$ là các số thực dương thoả mãn $x+y+z=1$. Chứng minh rằng:
$$\dfrac{x^4}{x^3+y^2+z^2}+\dfrac{y^4}{y^3+z^2+x^2}+\dfrac{z^4}{z^3+x^2+y^2}\ge \dfrac{1}{7}.$$

Bài 2. Tìm tất cả các hàm số $f:N^* \rightarrow  N^*$ thoả mãn đồng thời các điều kiện:

i)  $ f(mn)=f(m)f(n) \forall m,n \in N^* $.
ii) $f(m)+f(n)$ chia hết cho $m+n$, $\forall m,n \in N^* $.
iii) $f(2017)=2017^3$.

Bài 3.  Cho đường tròn $(O)$ và dây cung $AB$ cố định. $C$ là một điểm thay đổi trên cung lớn $AB$ sao cho tam giác $ABC$ nhọn. Gọi $I,I_a,I_b$ lần lượt là tâm đường tròn nội tiếp, tâm đường tròn bàng tiếp $\angle BAC$ và $\angle ABC$ của tam giác $ABC$.
a) Gọi $M$ đối xứng với $I$ qua $O$. Chứng minh rằng tam giác $MI_{a}I_{b}$ cân.
b) Gọi $H,K$ lần lượt là hình chiếu của $I_a,I_b$ trên $OI$. Đường thẳng qua $H$ vuông góc với $BI_a$ và đường thẳng qua $K$ vuông góc với $AI_b$ cắt nhau tại $P$. Chứng minh rằng $P$ thuộc một đường cố định khi $C$ thay đổi.

Bài 4.  Cho $S$ là tập hợp khác rỗng và $A_1,A_2,\ldots,A_m\ (m\ge 2)$ là $m$ tập con của $S$. Gọi $\mathcal T$ là tập hợp gồm tất cả các tập hợp $A_i\Delta A_j\ (1\le i,j \le m$). Chứng minh rằng $|\mathcal T| \ge m$. \medskip

(Ký hiệu $A\Delta B=(A\backslash B)\cup (B\backslash A)$ là hiệu đối xứng của hai tập hợp $A,B$).

 

Đề và đáp thi chọn đội dự tuyển PTNK năm học 2017 – 2018

Bài 1. Tìm tất cả các hàm số $f:\mathbb R \rightarrow \mathbb R $ thỏa mãn:
$$f(3f(x)+2y)=10x+f(f(y)+x),\ \forall x,y \in \mathbb R.$$

Bài 2.  Cho tam giác $ABC$ nhọn. Các điểm $D,E$ thay đổi trên cạnh $BC$ sao cho $\angle BAD = \angle CAE$ và $D$ nằm giữa $B,E$. Đường tròn ngoại tiếp các tam giác $ABD,ACE$ cắt nhau tại điểm $M$ khác $A$.
a)  Chứng minh rằng phân giác góc $\angle DME$ luôn đi qua một điểm cố định.
b) Gọi $I$ và $K$ lần lượt là tâm đường tròn nội tiếp của các tam giác $ABM,ACM$. Chứng minh rằng đường thẳng $IK$ luôn đi qua một điểm cố định.

Bài 3.  Cho $n\ge 3$ là số nguyên dương và $2n$ số thực dương $x_1,x_2,\ldots,x_n,y_1,y_2,\ldots,y_n$ thỏa mãn đồng thời các điều kiện sau:
i) $0< x_1y_1<x_2y_2<\ldots< x_ny_n$.
ii) $x_1+x_2+\cdots+x_k \ge y_1+y_2+\cdots+y_k\ \forall k \in {1,2,\ldots,n }$.

Chứng minh rằng $\dfrac{1}{x_1}+\dfrac{1}{x_2}+\ldots+\dfrac{1}{x_n} \le \dfrac{1}{y_1}+\dfrac{1}{y_2}+\ldots+\dfrac{1}{y_n}$.

Bài 4. Cho $S$ là tập hợp khác rỗng có hữu hạn phần tử. Kí hiệu $P(S)$ là tập hợp tất cả các tập con của $S$. Giả sử $f: P(S) \rightarrow P(S)$ là ánh xạ có tính chất sau: với mọi $X,Y \in P(S)$, nếu $X \subset Y$ thì $f(X) \subset f(Y)$.

Chứng minh rằng có tập hợp $T \in P(S)$ để $f(T) = T$.

Giải

Bài 1. 

Thay $y=-\frac{2f(x)}{3}$, ta có
$$f(0)=10x+f\left( f\left( -\frac{2f(x)}{3} \right)+x \right)$$
nên dễ thấy rằng $f$ toàn ánh vì $f(0)-10x$ nhận giá trị trên $\mathbb{R}.$
Giả sử tồn tại $a,b\in \mathbb{R}$ sao cho $f(a)=f(b).$ Thay $y$ lần lượt bởi $a,b,$ ta có
$$f(3f(x)+2a)=f(3f(y)+2b).$$
Vì tính toàn ánh nên có thể thay $3f(x)\to x$, tức là $f(x+2a)=f(x+2b)$ nên $f$ tuần hoàn chu kỳ $T=2(a-b).$ Khi đó, ta có $f(x)=f(x+T),\forall x\in \mathbb{R}.$

Trong đề bài, thay $x\to x+T$ thì
$f(3f(x)+2y)=10x+10T+f(2f(y)-x)$ nên $T=0.$ Suy ra $f$ đơn ánh. Cuối cùng, cho $x=0$ thì
$f(3f(0)+2y)=f(f(y))$ nên
$$3f(0)+2y=f(y)\Leftrightarrow f(y)=2y+\frac{3}{2}f(0),\forall y.$$
Thay $y=0,$ ta có ngay $f(0)=0$ nên $f(y)=2y.$ Thử lại ta thấy thỏa.

Vậy hàm số $f(x)$ cần tìm là $f(x)=2x,\forall x.$

Bài 2.

(a) Do tứ giác $ABDM,ACEM$ nội tiếp nên $\angle DAB=\angle DMB,\angle EAC=\angle EMC$, mà $\angle DAB=\angle EAC$ nên ta có $\angle DMB=\angle EMC.$ Ta sẽ chứng minh bổ đề sau

Bổ đề (hệ thức Steiner) $\frac{DB}{DC}\cdot \frac{EB}{EC}=\frac{A{{B}^{2}}}{A{{C}^{2}}}$.

Thật vậy, kẻ đường tròn$(ADE)$ cắt $AB,AC$ tại $X,Y.$ Khi đó, ta có $DX=EY$ (vì cùng chắn các cung bằng nhau), suy ra $XY\parallel DE$.
Áp dụng phương tích từ các điểm $B,C$ đến đường tròn $(ADE)$ thì
$$BD\cdot BE=BX\cdot BA \text{ và } CE\cdot CD=CY\cdot CA$$
nên suy ra $$\frac{BD\cdot BE}{CE\cdot CD}=\frac{AB}{AC}\cdot \frac{BX}{CY}=\frac{A{{B}^{2}}}{A{{C}^{2}}}.$$
Áp dụng bổ đề này vào tam giác $BMC$ với hai điểm $D,E.$ Ta cũng có $$\frac{DB}{DC}\cdot \frac{EB}{EC}=\frac{M{{B}^{2}}}{M{{C}^{2}}}.$$ Từ đó suy ra $\frac{MB}{MC}=\frac{AB}{AC}$. Gọi $MS$ là phân giác của $\angle DME$ với $S\in BC.$ Suy ra $MS$ cũng là phân giác của góc $\angle BMC.$ Do đó $$\frac{SB}{SC}=\frac{MB}{MC}=\frac{AB}{AC}$$ nên $S$ chính là chân đường phân giác góc $A$ của tam giác $ABC,$ là điểm cố định.

(b) Gọi $J$ là tâm nội tiếp tam giác $ABC$ thì rõ ràng $I\in BJ,K\in CJ.$
Đặt $\angle DAB=\angle EAC=2\alpha ,\angle DAE=2\beta $ thì
$$\frac{IB}{IJ}=\frac{{{S}_{IAB}}}{{{S}_{IAJ}}}=\frac{AI\cdot AB\cdot \sin \alpha }{AI\cdot AJ\cdot \sin \beta }=\frac{AB}{AJ}\cdot \frac{\sin \alpha }{\sin \beta }.$$
Tương tự thì $$\frac{KC}{JC}=\frac{AC}{AJ}\cdot \frac{\sin \alpha }{\sin \beta }$$ nên $\frac{IB}{IJ}:\frac{KC}{KJ}=\frac{AB}{AC}$. Từ đây gọi $T$ là giao điểm của $IK,BC$ thì theo định lý Menelaus cho tam giác $JBC,$ ta có $\frac{TB}{TC}=\frac{AB}{AC}$ nên $T$ là chân phân giác ngoài góc $A$ của tam giác $ABC,$ là điểm cố định.

 

Bài 3. 

Nhắc lại về khai triển Abel, xem như bổ đề:

Bổ đề. Xét 2 dãy số thực ${{a}_{1}},{{a}_{2}},\ldots ,{{a}_{n}}$ và ${{b}_{1}},{{b}_{2}},\ldots ,{{b}_{n}}$. Đặt ${{S}_{k}}={{b}_{1}}+{{b}_{2}}+\cdots +{{b}_{k}}$. Khi đó
$${{a}_{1}}{{b}_{1}}+{{a}_{2}}{{b}_{2}}+\cdots +{{a}_{n}}{{b}_{n}}=({{a}_{1}}-{{a}_{2}}){{S}_{1}}+({{a}_{2}}-{{a}_{3}}){{S}_{2}}+\cdots +({{a}_{n-1}}-{{a}_{n}}){{S}_{n}}+{{a}_{n}}{{S}_{n}}.$$
Trở lại bài toán đã cho, chuyển vế và quy đồng, ta cần có
$$\frac{{{x}_{1}}-{{y}_{1}}}{{{x}_{1}}{{y}_{1}}}+\frac{{{x}_{2}}-{{y}_{2}}}{{{x}_{2}}{{y}_{2}}}+\cdots +\frac{{{x}_{n}}-{{y}_{n}}}{{{x}_{n}}{{y}_{n}}}>0.$$
Đặt ${{b}_{k}}={{x}_{k}}-{{y}_{k}}$ và ${{a}_{k}}=\frac{1}{{{x}_{k}}{{y}_{k}}}$ với $1\le k\le n$, ta cần chứng minh
$${{a}_{1}}{{b}_{1}}+{{a}_{2}}{{b}_{2}}+\cdots +{{a}_{n}}{{b}_{n}}>0.$$
Chú ý rằng $${{S}_{k}}={{b}_{1}}+{{b}_{2}}+\cdots +{{b}_{k}}=({{x}_{1}}+{{x}_{2}}+\cdots +{{x}_{k}})-({{y}_{1}}+{{y}_{2}}+\cdots +{{y}_{k}})>0$$ đúng theo ii).
Ngoài ra, dãy ${{a}_{k}}$ là dãy giảm nên ${{a}_{1}}-{{a}_{2}},{{a}_{2}}-{{a}_{3}},\ldots ,{{a}_{n-1}}-{{a}_{n}}>0$. Từ đó, áp dụng khai triển Abel ở trên, ta có ngay đpcm.

 

Bài 4.

Nếu như $f(S)=S$ thì ta có đpcm.

Giả sử rằng $f(S)\ne S$. Ta đặt $f(S)={{S}_{1}}$ là một tập con thực sự của $S.$ Khi đó vì ${{S}_{1}}\subset S$ nên ta phải có $f({{S}_{1}})\subset f(S)\Rightarrow f({{S}_{1}})\subset {{S}_{1}}$.

Nếu $f({{S}_{1}})={{S}_{1}}$ thì ta cũng có đpcm nên giả sử $f({{S}_{1}})={{S}_{2}}\ne {{S}_{1}}$ và ${{S}_{2}}\subset {{S}_{1}}.$

Tiếp tục như thế, ta thấy rằng với mỗi số nguyên dương $k$ thì hoặc là $f({{S}_{k}})={{S}_{k}}$ hoặc $f({{S}_{k}})={{S}_{k+1}}$ là tập con thực sự của ${{S}_{k}}.$ Và nếu như không có trường hợp thứ nhất xảy ra thì quá trình này lặp lại vô hạn lần, và sinh ra vô hạn tập con thực sự của tập hữu hạn $S$ ban đầu. Đây là điều vô lý.

Vậy nên luôn tồn tại $T \in P(S)$ để cho $f(T)=T.$

Đề và lời giải thi chọn đội dự tuyển năm học 2018-2019

Bài 1. Tìm tất cả các hàm số $f:\mathbb R\rightarrow \mathbb R$ thoả mãn:
i) $f(-x)=-f(x)\ \forall x\in \mathbb R$.
ii) $f(f(x)-y)=2x+f(f(y+x))\ \forall x,y\in \mathbb R$.

Bài 2. Tìm tất cả các bộ số tự nhiên $(a,b,c)$ để $a^2+2b+c,b^2+2c+a,c^2+2a+b$ đều là các số chính phương.

Bài 3. Cho tập hợp $X={1,2,\ldots,396}$. Gọi $S_1,S_2,\ldots,S_k$ là $k$ tập con khác nhau của $X$ thoả mãn đồng thời hai điều kiện sau:

i)$|S_1|=|S_2|=\ldots=|S_k|=198$.
ii) $|S_i\cap S_j|\le 99\ \forall i,j\in \mathbb N^*, 1\le i<j\le k$.

Chứng minh rằng $k\le 6^{50}$.

Bài 4. Cho tam giác $ABC$ nhọn. Đường tròn thay đổi qua $B,C$ cắt các cạnh $AB,AC$ lần lượt tại $D,E$.

a) Gọi $H,K$ lần lượt là hình chiếu của $B$ trên $CD$ và $DE$. Chứng minh rằng $HK$ luôn đi qua một điểm cố định.
b) Gọi $Q$ là hình chiếu của $C$ trên $DE$. Đường tròn ngoại tiếp tam giác $BDK$ cắt $BC$ tại $M$, đường tròn ngoại tiếp tam giác $CEQ$ cắt $BC$ tại $N$. $KM,QN$ cắt nhau tại $X$. Chứng minh rằng $X$ thuộc một đường thẳng cố định.

Lời giải

Bài 1.

Trong điều kiện $(ii),$ thay $x$ bởi $-x,$ ta được
$$
f(f(-x)-y)=-2x+f(f(y)-x)),
$$
hay
$$-f(f(x)+y)=-2x+f(f(y)-x),\text{ với mọi } x,y\in\mathbb{R}. (*) $$

Thay vai trò của $x$ và $y$ trong $(ii)$, ta có
$
f(f(y)-x)=2y+f(f(x)+y),\text{ với mọi }x,y\in\mathbb{R}.
$
Thay vào $(*)$, ta có
$$
-f(f(x)+y)=-2x+2y+f(f(x)+y),
$$
hay
$$
f(f(x)+y)=x-y,\text{ với mọi }x,y\in\mathbb{R}.
$$
Thay $y$ bởi $f(y),$ ta có
$$
f(f(x)+f(y))=x-f(y),\text{ với mọi }x,y\in\mathbb{R}.
$$
Đổi vai trò của $x,y$, ta thu được
$$
x-f(y)=y-f(x), \text{ tức là } f(x)=-x+c,\text{ với mọi }x\in\mathbb{R}.
$$
Thay vào đề bài, ta suy ra $c=0.$ Vậy hàm số cần tìm là $f(x)=-x.$

Bài 2.

Không mất tính tổng quát, ta giả sử $a=\min (a,b,c).$ Nếu $a=0$ thì ta có $2b+c,b^2+2c,c^2+b$ đều là các số chính phương.
Nếu như $b \le c$ thì $c^2 \le c^2+b \le c^2+c <(c+1)^2$ là số chính phương, kéo theo $c^2+b=c^2$ nên $b=0$. Từ đây dễ dàng có $c=0$. Tương tự nếu $c \le b$ cũng có $b=c=0$. \medskip

Do đó, trong trường hợp này, ta có bộ nghiệm $(a,b,c)=(0,0,0)$.
Ta xét các trường hợp sau ứng với $a>0.$

  • Nếu $a\le b\le c.$ Khi đó $c^2< c^2+2a+b\le c^2+3c<(c+2)^2$. Do đó $c^2+2a+b=(c+1)^2,$ hay $2a+b=2c+1.$ Ta cũng có
    $$
    b^2 < b^2+2c+a = b^2+2a+b-1+a \leq b^2+4b – 1 < (b+2)^2,
    $$
    tức là
    $$
    b^2+2c+a=(b+1)^2,\ 2c+a=2b+1.
    $$
    Đẳng thức xảy ra khi $a=1,b=c$, từ đây dễ dàng tìm được $a=b=c=1$. Thử lại ta thấy bộ số này thỏa mãn.
  • Nếu $a \leq c \leq b$. Khi đó $b^2 < b^2+2c+a \leq b^2 + 3b < (b+2)^2$, tức $b^2+2c+a=(b+1)^2$ và $2c+a=2b+1$. Ta suy ra
    $$4a+2b = 4a+2c+a-1 \leq 7c – 1 < 8c+8.
    $$
    Do đó $2a+b < 4c+4$ và $c^2 < c^2+2a+b < (c+2)^2$. Do đó $2a+b=2c+1$. Kết hợp với $2c+a=2b+1$, ta suy ra
    $$
    (a,b,c)=(a,3a-2,\frac{5a-3}{2}).
    $$
    Do đó $a$ lẻ và trường hợp $a=1$ đã xét nên ở đây ta đặt $a=2t+1$, với $t \geq 1$. Khi đó $(a,b,c)=(2t+1,6t+1,5t+1)$. Vì $b^2+2c+a$ và $c^2+2a+b$ là các số chính phương nên ta xét điều kiện để $a^2+2b+c=4t^2+21t+4$ là số chính phương. Với $t \geq 3$, ta có
    $$
    (2t+4)^2 < 4t^2+21t+4<(2t+6)^2
    $$

Do đó $4t^2+21t+4 = (2t+5)^2$ và $t=21$. Như vậy $t \in {1,2,21}$. Thử trực tiếp, ta thấy chỉ có $t=21$ là thỏa mãn ứng với $(a,b,c)=(43,127,106)$.

Vậy tất cả bộ ba số thỏa mãn đề bài là $$(a,b,c)=(0,0,0),(1,1,1),(43,127,106).$$

Bài 3.

Vì $|S_i\cap S_j|\le 99$ với mọi $1\le i<j\le k$ nên mỗi bộ $100$ phần tử chỉ có thể được chứa tối đa trong $1$ tập hợp. Ta đếm các bộ $\{x_1,x_2,\ldots,x_{100},M \}$, trong đó $x_i\in X$ với mọi $i$ và $M$ là một trong các tập $S_i$, $M$ chứa $x_1,x_2,\ldots,x_{100}$.

  • Số cách chọn tập $M$ là $k$. Số cách chọn $100$ phần tử trong $M$ là $C^{100}_{198}.$
  • Số cách chọn $x_1,x_2,\ldots,x_{100}$ từ $X$ là $C^{100}_{396}.$ Với mỗi bộ $100$ phần tử như vậy, có tối đa $1$ tập $S_i$ thỏa mãn $S_i$ chứa $x_1,x_2,\ldots,x_{100}.$

Do đó ta có bất đẳng thức
$ kC^{100}{198} \le C^{100}{396} $
hay

$k \le \dfrac{C^{100}{396}}{C^{100}{198}}$

$=\dfrac{396!100!98!}{100!296!198!}$

$=\dfrac{396!98!}{198!296!} $

$=\dfrac{297\cdot 298 \ldots 396}{99\cdot 100\ldots 198}$
$=\dfrac{297\cdot 299\ldots 395}{99\cdot 100\ldots 148}\cdot\dfrac{298}{149}\cdot \dfrac{300}{150}\ldots \dfrac{396}{198}$
$\le 3^{50}\cdot 2^{50}=6^{50}.$

Ta có đpcm.

Bài 4.

(a) Gọi $F$ là giao điểm của $KH$ và $AC.$ Ta chứng minh $F$ cố định. Ta có tứ giác $BDEC$ nội tiếp nên $\angle BDC=\angle BEC.$ Tứ giác $KDHB$ cũng nội tiếp nên ta suy ra $\angle BDC=\angle BKF.$ \medskip

Do đó $\angle BEC=\angle BKF,$ tức là tứ giác $KEFB$ nội tiếp. Khi đó ta có $$\angle EFB=180^{\circ}-\angle BKE =90^{\circ}.$$ Do đó $BF\perp AC,$ tức là điểm $F$ cố định.

(b) Tứ giác $DKMB$ nội tiếp nên $\angle BMK=\angle KDB$. Ta suy ra
$$\angle NMX=\angle EDA.$$
Ta có $EQCN$ nội tiếp nên $$\angle QNC=\angle QEC, \text{ hay } \angle MNX=\angle AED.$$
Từ đó, ta suy ra $\triangle MNX\sim \triangle DEA.$ Gọi $G$ là chân đường cao từ $A$ đến $BC$ và $AG$ cắt $DE$ tại $P$. Khi đó $BC\perp AG$. Mà $BC\perp DM$ và $BC\perp EN$ nên
$$
AC \parallel DM \parallel EN.
$$

Do đó $\dfrac{DP}{PE}=\dfrac{MG}{GN}$. Mà $\triangle ADE\sim \triangle XMN$ nên $\angle XMN=\angle EPA.$ Mà $$\angle EPA=180^{\circ}-\angle PAC-\angle PEA$$ nên ta có
$$
\angle EPA=180^{\circ}-(90^{\circ}-\angle C)-B={\rm const}.
$$
Do đó $\angle XGN$ không đổi. Mà $G$ là điểm cố định nên $GX$ cố định. Như vậy $X$ di chuyển trên đường cố định.

 

Đáp án kì thi chọn đội dự tuyển PTNK năm học 2019 – 2020

Đề bài

Bài 1. Tìm giá trị nhỏ nhất của biểu thức
$$ P=\dfrac{a^4+b^4+2}{\left(a^2-a+1\right)\left(b^2-b+1\right)}, \text{ với } a,b \in \mathbb{R}. $$

Bài 2. Cho $\mathbb{Q^+}$ là tập hợp số hữu tỉ dương. Tìm tất cả các hàm $f:\mathbb{Q^+} \to \mathbb{Q^+}$ thỏa mãn
$$ f\left( {{x^2}f{{\left( y \right)}^2}} \right) = f{\left( x \right)^2}f\left( y \right), \text{ với mọi } x,y \in \mathbb{Q^+}. $$

Bài 3. Cho $x_1$, $x_2$, $x_3$, \dots là dãy số nguyên thỏa mãn đồng thời hai điều kiện
$$ 1=x_1<x_2<x_3 \dots \text{ và } x_{n+1}\leq 2n \text{ với } n=1,2,3 \dots $$
Chứng minh rằng với mọi số nguyên dương $k$, tồn tại các số nguyên $i>j$ sao cho $x_i-x_j = k.$

Bài 4. Cho tam giác $ABC$ cân tại $A$, nội tiếp đường tròn tâm $O$ bán kính $R$. Gọi $M$ là điểm trên cạnh $AB$ sao cho $\overrightarrow{AM}=\dfrac{1}{3}\overrightarrow{AB}.$ Đường tròn tâm $M$ bán kính $MB$ cắt đường tròn tâm $O$ tại điểm thứ hai là $D$. Một đường thẳng qua $M$ song song với $AD$ cắt $AC$ tại $N$. Chứng minh rằng $\overrightarrow{AN}=\dfrac{2}{3}\overrightarrow{AC}$.

Giải

Lời giải của nhóm các bạn NGUYỄN TĂNG VU, LÊ PHÚC LỮ, NGUYỄN TIẾN HOÀNG

Bài 1. 

Với mọi $x \in \mathbb{R}$, ta có
\[{x^4} + 1 – \frac{2}{9}{\left( {{x^2} – x + 1} \right)^2} = \frac{1}{9}{\left( {x + 1} \right)^2}\left( {7{x^2} – 10x + 7} \right) \geq 0. \]
Vì thế nên ta có
\[ P \ge \frac{2}{9}\frac{{{{\left( {{a^2} – a + 1} \right)}^2} + {{\left( {{b^2} – b + 1} \right)}^2}}}{{\left( {{a^2} – a + 1} \right)\left( {{b^2} – b + 1} \right)}} = \frac{2}{9}\left( {\frac{{{a^2} – a + 1}}{{{b^2} – b + 1}} + \frac{{{b^2} – b + 1}}{{{a^2} – a + 1}}} \right) \ge \frac{4}{9}. \]
Suy ra giá trị nhỏ nhất của $P$ là $\dfrac{4}{9}$, đạt được khi $a=b=-1.$

Bài 2.

Giả sử $f$ là một hàm thỏa mãn các yêu cầu của bài toán.
Đặt $f(1)=a>0$, trong phương trình đề cho, thay $x=y=1$ ta có $f(a^2)=a^3$. \medskip

Từ đó, tiếp tục lần lượt thay $x$ bởi $a^2$, $y$ bởi $1$ và $x$ bởi $1$, $y$ bởi $c^2$ vào phương trình ấy, ta thu được
\[ a^7 = f(a^6) = a^5. \]
Chú ý $a>0$ nên ta có $a=1$, tức $f(1)=1$. Thay $x$ bởi $1$ vào phương trình đề cho, ta có
\[ f\left( {f{{\left( y \right)}^2}} \right) = f\left( y \right), \text{ với mọi } y \in \mathbb{Q^+}. \]
Lại thay $y$ bởi $1$ vào phương trình đề cho, ta có
\[ f{\left( x \right)^2} = f\left( {{x^2}} \right), \text{ với mọi } x \in \mathbb{Q^+}. \]
Suy ra
\[ f\left( x \right) = f\left( {f{{\left( x \right)}^2}} \right) = f{\left( {f\left( x \right)} \right)^2} = \ldots = {f^{n + 1}}{\left( x \right)^{{2^n}}}, \text{ với mọi } x \in \mathbb{Q^+}, \]
trong đó $f^{n+1}(x)$ là $n+1$ lần tác động $f$ vào $x$. Từ đó, nếu tồn tại $q \in \mathbb{Q^+}$ sao cho tồn tại $p \in \mathbb{P}$ thỏa mãn $v_p(f(q)) \ne 0$ thì ta có
\[ {v_p}\left( {f\left( q \right)} \right) = {v_p}\left( {{f^{n + 1}}{{\left( q \right)}^{{2^n}}}} \right) = {2^n}{v_p}\left( {{f^{n + 1}}\left( q \right)} \right) \ne 0. \]
Trong đẳng thức trên, cho $n \to + \infty$ ta thấy điều vô lý. Suy ra $v_p(f(q)) = 0$ với mọi $q \in \mathbb{Q^+}$, $p \in \mathbb{P}$, hay $f(x) \equiv 1.$ \medskip

Thử lại, ta kết luận $f(x) \equiv 1$ là hàm duy nhất thỏa mãn yêu cầu bài toán.
\end{giai}

Bài 3. 

Với $k$ nguyên dương, ta xét $k+1$ số hạng của dãy là $x_1$, $x_2$, \dots, $x_{k+1}$. Ta có $x_1=1 \leq k$, gọi $q$ là số lớn nhất thỏa mãn $x_q \leq k$ thì ta có $q<k+1$ và
\[ 1 \leq x_1 < x_1 < \dots < x_q \leq k < x_{q+1}<\dots<x_{k+1}<2k. \]

Nếu tồn tại $1 \leq j < i \leq k+1 $ sao cho $x_i – x_j = k$ thì ta có ngay điều cần chứng minh. Ngược lại, ta có các số $$x_1+k,x_2+k, \dots x_q+k, x_{q+1}, \dots, x_{k+1}$$ là $k+1$ số nguyên đôi một phân biệt, tất cả đều lớn hơn $k$ nhưng lại không vượt quá $2k$, vô lí! \medskip

Từ đó suy ra với mọi $k$ nguyên dương,luôn tồn tại các số nguyên $i>j$ sao cho $x_i-x_j = k.$

Bài 4. 

Ta có $OB=OD$, $MB=MD$ nên dễ thấy $OM$ là phân giác ngoài của góc $AMD$, mà $OA=OD$ nên suy ra $O \in \left(AMD\right).$

Gọi $N’$ là giao điểm khác $A$ của $\left(AMD\right)$ và $AC$. Ta chứng minh $N$ trùng $N’$. \medskip

Thật vậy, ta có $\overrightarrow{AM}=\dfrac{1}{3}\overrightarrow{AB}$ nên $\angle{AMO}$ tù, do đó nếu $N’$ nằm ngoài tia $AC$ thì $N’$ nằm khác phía $O$ so với $AM$ nên $$\angle{AMO}=\angle{AN’O}=\angle{CAO}-\angle{AON’}<\angle{CAO}<90^\circ,$$ vô lý. Suy ra $N’$ nằm trên tia $AC$, kéo theo $AO$ là phân giác trong góc $MAN’$ nên $OM=ON’$, mà $OA=OD$ nên $MN’$ song song $AD$, suy ra $N$ trùng $N’$. \medskip

Từ đó, dễ thấy $AMND$ là hình thang cân nên $AN=MD=MB$, hơn nữa $N$ nằm trên tia $AC$ nên ta thu được $$\overrightarrow{AN}=\dfrac{2}{3}\overrightarrow{AC}.$$ Ta có điều cần chứng minh.